English
English, 03.03.2021 20:20, StephenCurry34

What effect does the use of the logical fallacy in this paragraph have on the credibility of the author’s argument? Answer choices for the above question

A. It strengthens the argument because it shows that the author carefully researched the results.

B. It strengthens the argument because it shows that the author has narrowed the potential conclusions.

C. It weakens the argument because it shows that the author may have failed to consider all outcomes.

D. It weakens the argument because it shows that the author has based the argument on an incorrect idea.

answer
Answers: 1

Other questions on the subject: English

image
English, 22.06.2019 04:30, singlegirlforlife541
On a scale 1 to 10 how responsible are you? explain your answer
Answers: 1
image
English, 22.06.2019 05:00, mshurney
How did the english teacher ponyboy? the outsiders
Answers: 1
image
English, 22.06.2019 06:20, jae222
Which two lines in this excerpt from arthur conan doyle's "the contest" suggest that emperor nero was ruthless? each choice is in brackets. [an hour later the shepherd was well on his way to his mountain home, and about the same time the emperor, having received the chaplet of olympia for the incomparable excellence of his performance, was making inquiries with a frowning brow as to who the insolent person might be who had dared to utter such contemptuous criticisms.] [“bring him to me here this instant,” said he, “and let marcus with his knife and branding-iron be in attendance.”] [“rumours! ” cried the angry nero. “what do you mean, arsenius? i tell you that the fellow was an ignorant upstart, with the bearing of a boor and the voice of a peacock.] [i have half a mind to burn their town about their ears so that they may remember my visit.”] [“i conquered! you are mad, arsenius. what do you mean? ”] [it is whispered that for once the great god pan has condescended to measure himself against a mortal.”]
Answers: 2
image
English, 22.06.2019 08:00, mandy9386
Which function is positive for the entire interval (-3, -2]?
Answers: 3
Do you know the correct answer?
What effect does the use of the logical fallacy in this paragraph have on the credibility of the aut...

Questions in other subjects:

Konu
Mathematics, 02.12.2020 22:40